Difference between revisions of "2009 AMC 10A Problems/Problem 7"

(See also)
Line 3: Line 3:
  
 
<math>
 
<math>
\mathrm{(A)}\ \frac{12}{5}
+
\mathrm{(A)}\ \frac{12}{5} \qquad
\qq\ 42
+
\mathrm{(B)}\ 3 \qquad
 +
\mathrm{(C)} \frac{10}{3} \qquad
 +
\mathrm{(D)} 38 \qquad
 +
\mathrm{(E)} 42 \qquad
 
</math>
 
</math>
  
=
+
 
 +
==Solution==

Revision as of 12:39, 18 April 2021

Problem

A carton contains milk that is $2$% fat, an amount that is $40$% less fat than the amount contained in a carton of whole milk. What is the percentage of fat in whole milk?

$\mathrm{(A)}\ \frac{12}{5} \qquad \mathrm{(B)}\ 3 \qquad \mathrm{(C)} \frac{10}{3} \qquad \mathrm{(D)} 38 \qquad \mathrm{(E)} 42 \qquad$


Solution